LSAT and Law School Admissions Forum

Get expert LSAT preparation and law school admissions advice from PowerScore Test Preparation.

 Administrator
PowerScore Staff
  • PowerScore Staff
  • Posts: 8919
  • Joined: Feb 02, 2011
|
#81497
Complete Question Explanation

Flaw in the Reasoning. The correct answer choice is (C).

Answer choice (A):

Answer choice (B):

Answer choice (C): This is the correct answer choice.

Answer choice (D):

Answer choice (E):

This explanation is still in progress. Please post any questions below!
 onlywinter
  • Posts: 17
  • Joined: Apr 18, 2018
|
#59323
Hi,

I chose C but viewed E as a contender. Since the stimulus from a discussion of the dofferences in effectiveness to an exaggerated conclusion unsupported by the premises ("There is no established medical reason"), I chose C. It seemed a more fundamental flaw than E. However, I would contend that E could be a flaw, too. We learn that "drug Z is either no more or only slightly more effective than drug Y", couldn't we say that the author is "confusing economic reasons for selecting a treatment with medical reasons"? If drug Z is even slightly more effective but the writer is attacking the drug for being more expensive, would it not be true that the author is taking more into account economic than medical reasons?
 Ben DiFabbio
PowerScore Staff
  • PowerScore Staff
  • Posts: 39
  • Joined: Aug 02, 2018
|
#59483
onlywinter wrote:Hi,

I chose C but viewed E as a contender. Since the stimulus from a discussion of the dofferences in effectiveness to an exaggerated conclusion unsupported by the premises ("There is no established medical reason"), I chose C. It seemed a more fundamental flaw than E. However, I would contend that E could be a flaw, too. We learn that "drug Z is either no more or only slightly more effective than drug Y", couldn't we say that the author is "confusing economic reasons for selecting a treatment with medical reasons"? If drug Z is even slightly more effective but the writer is attacking the drug for being more expensive, would it not be true that the author is taking more into account economic than medical reasons?
Hey Winter,

I can see what enticed you about answer choice E, since the stimulus does mention that drug Z is more expensive than drug Y. However, when you look back at the conclusion, you'll see that it does not actually urge doctors to recommend drug Y over drug Z because drug Y is cheaper. The conclusion just says that "there is no established medical reason for doctors to use drug Z rather than drug Y." The conclusion is focused on the merits of drug Z.

The flaw in the argument, which answer choice C points out, is that there might be some medical reasons to choose Z over Y that have nothing to do with either drug's effectiveness. For example, maybe Y has some crazy side effects that Z doesn't have.

I hope that helps, and happy prepping!

- Ben

Get the most out of your LSAT Prep Plus subscription.

Analyze and track your performance with our Testing and Analytics Package.